Difference between revisions of "1995 AJHSME Problems/Problem 5"

(Created page with "==Problem== Find the smallest whole number that is larger than the sum <cmath>2\dfrac{1}{2}+3\dfrac{1}{3}+4\dfrac{1}{4}+5\dfrac{1}{5}.</cmath> <math>\text{(A)}\ 14 \qquad \tex...")
 
Line 7: Line 7:
 
<math>\text{(A)}\ 14 \qquad \text{(B)}\ 15 \qquad \text{(C)}\ 16 \qquad \text{(D)}\ 17 \qquad \text{(E)}\ 18</math>
 
<math>\text{(A)}\ 14 \qquad \text{(B)}\ 15 \qquad \text{(C)}\ 16 \qquad \text{(D)}\ 17 \qquad \text{(E)}\ 18</math>
  
==Solution 1==
+
==Solution==
 +
===Solution 1===
  
 
Adding the whole numbers gives <math>2 + 3 + 4 + 5 = 14</math>.
 
Adding the whole numbers gives <math>2 + 3 + 4 + 5 = 14</math>.
Line 13: Line 14:
 
Adding the fractions gives <math>\frac{1}{2} + \frac{1}{3} + \frac{1}{4} + \frac{1}{5} = \frac{30}{60} + \frac{20}{60} + \frac{15}{60} + \frac{12}{60} = \frac{77}{60}</math>.  This will create one more whole, and a fraction that is less than <math>1</math>.  Thus, the smalleset whole number that is less than <math>15</math> plus some fractional part is <math>16</math>, and the answer is <math>\boxed{C}</math>.
 
Adding the fractions gives <math>\frac{1}{2} + \frac{1}{3} + \frac{1}{4} + \frac{1}{5} = \frac{30}{60} + \frac{20}{60} + \frac{15}{60} + \frac{12}{60} = \frac{77}{60}</math>.  This will create one more whole, and a fraction that is less than <math>1</math>.  Thus, the smalleset whole number that is less than <math>15</math> plus some fractional part is <math>16</math>, and the answer is <math>\boxed{C}</math>.
  
==Solution 2==
+
===Solution 2===
  
 
Convert the fractional parts to decimals, and approximate the answer.  <math>2.5 + 3.33 + 4.25 + 5.2 = 15.28</math>, and the answer is <math>16</math>, which is <math>\boxed{C}</math>.  
 
Convert the fractional parts to decimals, and approximate the answer.  <math>2.5 + 3.33 + 4.25 + 5.2 = 15.28</math>, and the answer is <math>16</math>, which is <math>\boxed{C}</math>.  

Revision as of 02:57, 23 December 2012

Problem

Find the smallest whole number that is larger than the sum

\[2\dfrac{1}{2}+3\dfrac{1}{3}+4\dfrac{1}{4}+5\dfrac{1}{5}.\]

$\text{(A)}\ 14 \qquad \text{(B)}\ 15 \qquad \text{(C)}\ 16 \qquad \text{(D)}\ 17 \qquad \text{(E)}\ 18$

Solution

Solution 1

Adding the whole numbers gives $2 + 3 + 4 + 5 = 14$.

Adding the fractions gives $\frac{1}{2} + \frac{1}{3} + \frac{1}{4} + \frac{1}{5} = \frac{30}{60} + \frac{20}{60} + \frac{15}{60} + \frac{12}{60} = \frac{77}{60}$. This will create one more whole, and a fraction that is less than $1$. Thus, the smalleset whole number that is less than $15$ plus some fractional part is $16$, and the answer is $\boxed{C}$.

Solution 2

Convert the fractional parts to decimals, and approximate the answer. $2.5 + 3.33 + 4.25 + 5.2 = 15.28$, and the answer is $16$, which is $\boxed{C}$.

See Also

1995 AJHSME (ProblemsAnswer KeyResources)
Preceded by
Problem 4
Followed by
Problem 6
1 2 3 4 5 6 7 8 9 10 11 12 13 14 15 16 17 18 19 20 21 22 23 24 25
All AJHSME/AMC 8 Problems and Solutions